LSAT and Law School Admissions Forum

Get expert LSAT preparation and law school admissions advice from PowerScore Test Preparation.

 Cmax979
  • Posts: 1
  • Joined: Jul 17, 2015
|
#19122
I wanted to know if anyone could explain why answer choice C on the aforementioned question is incorrect it deals with the placement of trees in a botanical garden and it seems as though either C or E could work.
 Robert Carroll
PowerScore Staff
  • PowerScore Staff
  • Posts: 1787
  • Joined: Dec 06, 2013
|
#19123
C,

For this Must Be True question, you cannot be sure that answer choice (C) is true. Some sycamores are older than maples, which are all older than dogwoods, but there is no comparison possible between the sycamores that are not older than every maple, and these are the only sycamores that could possibly be younger than some dogwoods.

Think of it like this:

OS - old sycamores (that is, the majority of sycamores older than any maple)

YS - young sycamores (every sycamore not in the old sycamores)

M - maples

D - dogwoods

You know that OS > M > D. You do not know where to place YS. It's somewhere younger than the oldest maple, but how far? Since this is unknown, you cannot be sure to the degree of proof needed in a Must Be True question.

Since T (tulips) > M, but some of the YS have to be younger than the oldest maple, you know what answer choice (E) is saying is necessarily true.

Robert Carroll

Get the most out of your LSAT Prep Plus subscription.

Analyze and track your performance with our Testing and Analytics Package.